Difference between revisions of "1988 AHSME Problems"

m
 
(8 intermediate revisions by one other user not shown)
Line 1: Line 1:
 +
{{AHSME Problems
 +
|year = 1988
 +
}}
 
==Problem 1==
 
==Problem 1==
 
<math>\sqrt{8}+\sqrt{18}= </math>
 
<math>\sqrt{8}+\sqrt{18}= </math>
Line 22: Line 25:
  
 
==Problem 3==
 
==Problem 3==
 +
 +
<asy>
 +
draw((0,0)--(1,0)--(1,4)--(0,4)--(0,0)--(0,1)--(-1,1)--(-1,2));
 +
draw((-1,2)--(0,2)--(0,4)--(-1,4)--(-1,5)--(1,5)--(1,6)--(0,6));
 +
draw((0,6)--(0,5)--(3,5)--(3,6)--(4,6)--(4,2)--(5,2));
 +
draw((5,2)--(5,1)--(1,1)--(3,1)--(3,0)--(4,0)--(4,1));
 +
draw((1,4)--(3,4)--(3,2)--(1,2)--(4,2)--(3,2)--(3,6));
 +
draw((3,6)--(4,6)--(4,5)--(5,5)--(5,4)--(4,4));
 +
</asy>
  
 
Four rectangular paper strips of length <math>10</math> and width <math>1</math> are put flat on a table and overlap perpendicularly as shown. How much area of the table is covered?
 
Four rectangular paper strips of length <math>10</math> and width <math>1</math> are put flat on a table and overlap perpendicularly as shown. How much area of the table is covered?
  
<asy>
+
<math>\text{(A)}\ 36 \qquad  
draw((0,0)--(1,0)--(1,4)--(0,4)--(0,0)--(0,1)--(-1,1)--(-1,2)--(0,2)--(0,4)--(-1,4)--(-1,5)--(1,5)--(1,6)--(0,6));
 
draw((0,6)--(0,5)--(3,5)--(3,6)--(4,6)--(4,2)--(5,2)--(5,1)--(1,1)--(3,1)--(3,0)--(4,0)--(4,1));
 
draw((1,4)--(3,4)--(3,2)--(1,2)--(4,2)--(3,2)--(3,6)--(4,6)--(4,5)--(5,5)--(5,4)--(4,4));
 
<\asy>
 
 
 
$\text{(A)}\ 36 \qquad  
 
 
\text{(B)}\ 40 \qquad  
 
\text{(B)}\ 40 \qquad  
 
\text{(C)}\ 44 \qquad  
 
\text{(C)}\ 44 \qquad  
 
\text{(D)}\ 98 \qquad  
 
\text{(D)}\ 98 \qquad  
\text{(E)}\ 100 $    
+
\text{(E)}\ 100</math>    
 
    
 
    
 
[[1988 AHSME Problems/Problem 3|Solution]]
 
[[1988 AHSME Problems/Problem 3|Solution]]
 +
  
 
==Problem 4==
 
==Problem 4==
  
The slope of the line $\frac{x}{3} + \frac{y}{2} = 1$ is
+
The slope of the line <math>\frac{x}{3} + \frac{y}{2} = 1</math> is
  
$\textbf{(A)}\ -\frac{3}{2}\qquad
+
<math>\textbf{(A)}\ -\frac{3}{2}\qquad
 
\textbf{(B)}\ -\frac{2}{3}\qquad
 
\textbf{(B)}\ -\frac{2}{3}\qquad
 
\textbf{(C)}\ \frac{1}{3}\qquad
 
\textbf{(C)}\ \frac{1}{3}\qquad
 
\textbf{(D)}\ \frac{2}{3}\qquad
 
\textbf{(D)}\ \frac{2}{3}\qquad
\textbf{(E)}\ \frac{3}{2}$    
+
\textbf{(E)}\ \frac{3}{2}</math>    
 
    
 
    
 
[[1988 AHSME Problems/Problem 4|Solution]]
 
[[1988 AHSME Problems/Problem 4|Solution]]
Line 53: Line 60:
 
==Problem 5==
 
==Problem 5==
  
If $b$ and $c$ are constants and $(x + 2)(x + b) = x^2 + cx + 6$, then $c$ is
+
If <math>b</math> and <math>c</math> are constants and <math>(x + 2)(x + b) = x^2 + cx + 6</math>, then <math>c</math> is
  
$\textbf{(A)}\ -5\qquad
+
<math>\textbf{(A)}\ -5\qquad
 
\textbf{(B)}\ -3\qquad
 
\textbf{(B)}\ -3\qquad
 
\textbf{(C)}\ -1\qquad
 
\textbf{(C)}\ -1\qquad
 
\textbf{(D)}\ 3\qquad
 
\textbf{(D)}\ 3\qquad
\textbf{(E)}\ 5$
+
\textbf{(E)}\ 5</math>
 
    
 
    
 
[[1988 AHSME Problems/Problem 5|Solution]]
 
[[1988 AHSME Problems/Problem 5|Solution]]
Line 67: Line 74:
 
A figure is an equiangular parallelogram if and only if it is a
 
A figure is an equiangular parallelogram if and only if it is a
  
$\textbf{(A)}\ \text{rectangle}\qquad
+
<math>\textbf{(A)}\ \text{rectangle}\qquad
 
\textbf{(B)}\ \text{regular polygon}\qquad
 
\textbf{(B)}\ \text{regular polygon}\qquad
 
\textbf{(C)}\ \text{rhombus}\qquad
 
\textbf{(C)}\ \text{rhombus}\qquad
 
\textbf{(D)}\ \text{square}\qquad
 
\textbf{(D)}\ \text{square}\qquad
\textbf{(E)}\ \text{trapezoid}$  
+
\textbf{(E)}\ \text{trapezoid}</math>  
  
 
[[1988 AHSME Problems/Problem 6|Solution]]
 
[[1988 AHSME Problems/Problem 6|Solution]]
Line 77: Line 84:
 
==Problem 7==
 
==Problem 7==
  
Estimate the time it takes to send $60$ blocks of data over a communications channel if each block consists of $512$
+
Estimate the time it takes to send <math>60</math> blocks of data over a communications channel if each block consists of <math>512</math>
"chunks" and the channel can transmit $120$ chunks per second.
+
"chunks" and the channel can transmit <math>120</math> chunks per second.
  
$\textbf{(A)}\ 0.04 \text{ seconds}\qquad
+
<math>\textbf{(A)}\ 0.04 \text{ seconds}\qquad
 
\textbf{(B)}\ 0.4 \text{ seconds}\qquad
 
\textbf{(B)}\ 0.4 \text{ seconds}\qquad
 
\textbf{(C)}\ 4 \text{ seconds}\qquad
 
\textbf{(C)}\ 4 \text{ seconds}\qquad
 
\textbf{(D)}\ 4\text{ minutes}\qquad
 
\textbf{(D)}\ 4\text{ minutes}\qquad
\textbf{(E)}\ 4\text{ hours}$
+
\textbf{(E)}\ 4\text{ hours}</math>
 
    
 
    
 
[[1988 AHSME Problems/Problem 7|Solution]]
 
[[1988 AHSME Problems/Problem 7|Solution]]
Line 90: Line 97:
 
==Problem 8==
 
==Problem 8==
  
If $\frac{b}{a} = 2$ and $\frac{c}{b} = 3$, what is the ratio of $a + b$ to $b + c$?
+
If <math>\frac{b}{a} = 2</math> and <math>\frac{c}{b} = 3</math>, what is the ratio of <math>a + b</math> to <math>b + c</math>?
  
$\textbf{(A)}\ \frac{1}{3}\qquad
+
<math>\textbf{(A)}\ \frac{1}{3}\qquad
 
\textbf{(B)}\ \frac{3}{8}\qquad
 
\textbf{(B)}\ \frac{3}{8}\qquad
 
\textbf{(C)}\ \frac{3}{5}\qquad
 
\textbf{(C)}\ \frac{3}{5}\qquad
 
\textbf{(D)}\ \frac{2}{3}\qquad
 
\textbf{(D)}\ \frac{2}{3}\qquad
\textbf{(E)}\ \frac{3}{4}  $    
+
\textbf{(E)}\ \frac{3}{4}  </math>    
 
    
 
    
 
[[1988 AHSME Problems/Problem 8|Solution]]
 
[[1988 AHSME Problems/Problem 8|Solution]]
Line 102: Line 109:
 
==Problem 9==
 
==Problem 9==
  
An $8$'\text{ X }$10$' table sits in the corner of a square room, as in Figure $1$ below.
+
<asy>
The owners desire to move the table to the position shown in Figure $2$.
+
defaultpen(linewidth(0.7)+fontsize(10));
The side of the room is $S$ feet. What is the smallest integer value of $S$ for which the table can be moved as desired without tilting it or taking it apart?
+
pair A=(0,0), B=(16,0), C=(16,16), D=(0,16), E=(32,0), F=(48,0), G=(48,16), H=(32,16), I=(0,8), J=(10,8), K=(10,16), L=(32,6), M=(40,6), N=(40,16);
 +
draw(A--B--C--D--A^^E--F--G--H--E^^I--J--K^^L--M--N);
 +
label("S", (18,8));
 +
label("S", (50,8));
 +
label("Figure 1", (A+B)/2, S);
 +
label("Figure 2", (E+F)/2, S);
 +
label("10'", (I+J)/2, S);
 +
label("8'", (12,12));
 +
label("8'", (L+M)/2, S);
 +
label("10'", (42,11));
 +
label("table", (5,12));
 +
label("table", (36,11));
 +
</asy>
  
defaultpen(linewidth(0.7)+fontsize(10));pair A=(0,0), B=(16,0), C=(16,16), D=(0,16), E=(32,0), F=(48,0), G=(48,16), H=(32,16)...
+
An <math>8' \times 10'</math> table sits in the corner of a square room, as in Figure <math>1</math> below.  
 +
The owners desire to move the table to the position shown in Figure <math>2</math>.  
 +
The side of the room is <math>S</math> feet. What is the smallest integer value of <math>S</math> for which the table can be moved as desired without tilting it or taking it apart?
  
$\textbf{(A)}\ 11\qquad
+
<math>\textbf{(A)}\ 11\qquad
 
\textbf{(B)}\ 12\qquad
 
\textbf{(B)}\ 12\qquad
 
\textbf{(C)}\ 13\qquad
 
\textbf{(C)}\ 13\qquad
 
\textbf{(D)}\ 14\qquad
 
\textbf{(D)}\ 14\qquad
\textbf{(E)}\ 15  $  
+
\textbf{(E)}\ 15  </math>  
 
    
 
    
 
[[1988 AHSME Problems/Problem 9|Solution]]
 
[[1988 AHSME Problems/Problem 9|Solution]]
Line 118: Line 139:
 
==Problem 10==
 
==Problem 10==
  
In an experiment, a scientific constant $C$ is determined to be $2.43865$ with an error of at most $\pm 0.00312$.  
+
In an experiment, a scientific constant <math>C</math> is determined to be <math>2.43865</math> with an error of at most <math>\pm 0.00312</math>.  
The experimenter wishes to announce a value for $C$ in which every digit is significant.  
+
The experimenter wishes to announce a value for <math>C</math> in which every digit is significant.  
That is, whatever $C$ is, the announced value must be the correct result when $C$ is rounded to that number of digits.  
+
That is, whatever <math>C</math> is, the announced value must be the correct result when <math>C</math> is rounded to that number of digits.  
The most accurate value the experimenter can announce for $C$ is
+
The most accurate value the experimenter can announce for <math>C</math> is
  
$\textbf{(A)}\ 2\qquad
+
<math>\textbf{(A)}\ 2\qquad
 
\textbf{(B)}\ 2.4\qquad
 
\textbf{(B)}\ 2.4\qquad
 
\textbf{(C)}\ 2.43\qquad
 
\textbf{(C)}\ 2.43\qquad
 
\textbf{(D)}\ 2.44\qquad
 
\textbf{(D)}\ 2.44\qquad
\textbf{(E)}\ 2.439  $  
+
\textbf{(E)}\ 2.439  </math>  
 
    
 
    
 
[[1988 AHSME Problems/Problem 10|Solution]]
 
[[1988 AHSME Problems/Problem 10|Solution]]
Line 133: Line 154:
 
==Problem 11==
 
==Problem 11==
  
On each horizontal line in the figure below, the five large dots indicate the populations of cities $A, B, C, D$ and $E$ in the year indicated.  
+
<asy>
Which city had the greatest percentage increase in population from $1970$ to $1980$?
+
defaultpen(linewidth(0.7)+fontsize(10));
 +
pair A=(5,0), B=(7,0), C=(10,0), D=(13,0), E=(16,0);
 +
pair F=(4,3), G=(5,3), H=(7,3), I=(10,3), J=(12,3);
 +
dot(A);
 +
dot(B);
 +
dot(C);
 +
dot(D);
 +
dot(E);
 +
dot(F);
 +
dot(G);
 +
dot(H);
 +
dot(I);
 +
dot(J);
 +
draw((0,0)--(18,0)^^(0,3)--(18,3));
 +
draw((0,0)--(0,.5)^^(5,0)--(5,.5)^^(10,0)--(10,.5)^^(15,0)--(15,.5));
 +
draw((0,3)--(0,2.5)^^(5,3)--(5,2.5)^^(10,3)--(10,2.5)^^(15,3)--(15,2.5));
 +
draw((1,0)--(1,.2)^^(2,0)--(2,.2)^^(3,0)--(3,.2)^^(4,0)--(4,.2)^^(6,0)--(6,.2)^^(7,0)--(7,.2)^^(8,0)--(8,.2)^^(9,0)--(9,.2)^^(10,0)--(10,.2)^^(11,0)--(11,.2)^^(12,0)--(12,.2)^^(13,0)--(13,.2)^^(14,0)--(14,.2)^^(16,0)--(16,.2)^^(17,0)--(17,.2)^^(18,0)--(18,.2));
 +
draw((1,3)--(1,2.8)^^(2,3)--(2,2.8)^^(3,3)--(3,2.8)^^(4,3)--(4,2.8)^^(6,3)--(6,2.8)^^(7,3)--(7,2.8)^^(8,3)--(8,2.8)^^(9,3)--(9,2.8)^^(10,3)--(10,2.8)^^(11,3)--(11,2.8)^^(12,3)--(12,2.8)^^(13,3)--(13,2.8)^^(14,3)--(14,2.8)^^(16,3)--(16,2.8)^^(17,3)--(17,2.8)^^(18,3)--(18,2.8));
 +
label("A", A, S);
 +
label("B", B, S);
 +
label("C", C, S);
 +
label("D", D, S);
 +
label("E", E, S);
 +
label("A", F, N);
 +
label("B", G, N);
 +
label("C", H, N);
 +
label("D", I, N);
 +
label("E", J, N);
 +
label("1970", (0,3), W);
 +
label("1980", (0,0), W);
 +
label("0", (0,1.5));
 +
label("50", (5,1.5));
 +
label("100", (10,1.5));
 +
label("150", (15,1.5));
 +
label("Population in thousands", (9,-3));
 +
</asy>
  
defaultpen(linewidth(0.7)+fontsize(10));pair A=(5,0), B=(7,0), C=(10,0), D=(13,0), E=(16,0);pair F=(4,3), G=(5,3), H=(7,3), I...
+
On each horizontal line in the figure below, the five large dots indicate the populations of cities <math>A, B, C, D</math> and <math>E</math> in the year indicated.  
 +
Which city had the greatest percentage increase in population from <math>1970</math> to <math>1980</math>?
  
$\textbf{(A)}\ A\qquad
+
<math>\textbf{(A)}\ A\qquad
 
\textbf{(B)}\ B\qquad
 
\textbf{(B)}\ B\qquad
 
\textbf{(C)}\ C\qquad
 
\textbf{(C)}\ C\qquad
 
\textbf{(D)}\ D\qquad
 
\textbf{(D)}\ D\qquad
\textbf{(E)}\ E    $
+
\textbf{(E)}\ E    </math>
 
    
 
    
 
[[1988 AHSME Problems/Problem 11|Solution]]
 
[[1988 AHSME Problems/Problem 11|Solution]]
Line 148: Line 205:
 
==Problem 12==
 
==Problem 12==
  
Each integer $1$ through $9$ is written on a separate slip of paper and all nine slips are put into a hat.  
+
Each integer <math>1</math> through <math>9</math> is written on a separate slip of paper and all nine slips are put into a hat.  
 
Jack picks one of these slips at random and puts it back. Then Jill picks a slip at random.  
 
Jack picks one of these slips at random and puts it back. Then Jill picks a slip at random.  
 
Which digit is most likely to be the units digit of the sum of Jack's integer and Jill's integer?
 
Which digit is most likely to be the units digit of the sum of Jack's integer and Jill's integer?
  
$\textbf{(A)}\ 0\qquad
+
<math>\textbf{(A)}\ 0\qquad
 
\textbf{(B)}\ 1\qquad
 
\textbf{(B)}\ 1\qquad
 
\textbf{(C)}\ 8\qquad
 
\textbf{(C)}\ 8\qquad
 
\textbf{(D)}\ 9\qquad
 
\textbf{(D)}\ 9\qquad
\textbf{(E)}\ \text{each digit is equally likely}    $
+
\textbf{(E)}\ \text{each digit is equally likely}    </math>
 
    
 
    
 
[[1988 AHSME Problems/Problem 12|Solution]]
 
[[1988 AHSME Problems/Problem 12|Solution]]
Line 162: Line 219:
 
==Problem 13==
 
==Problem 13==
  
If $\sin\ x\ =\ 3\ \cos\ x $ then what is $\sin\ x\ \cos\ x$?
+
If <math>\sin(x)= 3\cos(x)</math> then what is <math>\sin(x) \cdot \cos(x)</math>?
  
$\textbf{(A)}\ \frac{1}{6}\qquad
+
<math>\textbf{(A)}\ \frac{1}{6}\qquad
 
\textbf{(B)}\ \frac{1}{5}\qquad
 
\textbf{(B)}\ \frac{1}{5}\qquad
 
\textbf{(C)}\ \frac{2}{9}\qquad
 
\textbf{(C)}\ \frac{2}{9}\qquad
 
\textbf{(D)}\ \frac{1}{4}\qquad
 
\textbf{(D)}\ \frac{1}{4}\qquad
\textbf{(E)}\ \frac{3}{10} $    
+
\textbf{(E)}\ \frac{3}{10} </math>    
 
    
 
    
 
[[1988 AHSME Problems/Problem 13|Solution]]
 
[[1988 AHSME Problems/Problem 13|Solution]]
Line 176: Line 233:
 
For any real number a and positive integer k, define
 
For any real number a and positive integer k, define
  
${a \choose k} = \frac{a(a-1)(a-2)\cdots(a-(k-1))}{k(k-1)(k-2)\cdots(2)(1)}$
+
<math>{a \choose k} = \frac{a(a-1)(a-2)\cdots(a-(k-1))}{k(k-1)(k-2)\cdots(2)(1)}</math>
  
 
What is
 
What is
  
${-\frac{1}{2} \choose 100} \div {\frac{1}{2} \choose 100}$?
+
<math>{-\frac{1}{2} \choose 100} \div {\frac{1}{2} \choose 100}</math>?
  
$\textbf{(A)}\ -199\qquad
+
<math>\textbf{(A)}\ -199\qquad
 
\textbf{(B)}\ -197\qquad
 
\textbf{(B)}\ -197\qquad
 
\textbf{(C)}\ -1\qquad
 
\textbf{(C)}\ -1\qquad
 
\textbf{(D)}\ 197\qquad
 
\textbf{(D)}\ 197\qquad
\textbf{(E)}\ 199    $
+
\textbf{(E)}\ 199    </math>
 
    
 
    
 
[[1988 AHSME Problems/Problem 14|Solution]]
 
[[1988 AHSME Problems/Problem 14|Solution]]
Line 192: Line 249:
 
==Problem 15==
 
==Problem 15==
  
If $a$ and $b$ are integers such that $x^2 - x - 1$ is a factor of $ax^3 + bx^2 + 1$, then $b$ is
+
If <math>a</math> and <math>b</math> are integers such that <math>x^2 - x - 1</math> is a factor of <math>ax^3 + bx^2 + 1</math>, then <math>b</math> is
  
$\textbf{(A)}\ -2\qquad
+
<math>\textbf{(A)}\ -2\qquad
 
\textbf{(B)}\ -1\qquad
 
\textbf{(B)}\ -1\qquad
 
\textbf{(C)}\ 0\qquad
 
\textbf{(C)}\ 0\qquad
 
\textbf{(D)}\ 1\qquad
 
\textbf{(D)}\ 1\qquad
\textbf{(E)}\ 2$      
+
\textbf{(E)}\ 2</math>      
 
    
 
    
 
[[1988 AHSME Problems/Problem 15|Solution]]
 
[[1988 AHSME Problems/Problem 15|Solution]]
  
 
==Problem 16==
 
==Problem 16==
 
$ABC$ and $A'B'C'$ are equilateral triangles with parallel sides and the same center,
 
as in the figure. The distance between side $BC$ and side $B'C'$ is $\frac{1}{6}$ the altitude of $\triangle ABC$.
 
The ratio of the area of $\triangle A'B'C'$ to the area of $\triangle ABC$ is
 
  
 
<asy>
 
<asy>
Line 218: Line 271:
 
label("B", E, E);
 
label("B", E, E);
 
label("C", F, W);
 
label("C", F, W);
<\asy>
+
</asy>
  
 +
<math>ABC</math> and <math>A'B'C'</math> are equilateral triangles with parallel sides and the same center,
 +
as in the figure. The distance between side <math>BC</math> and side <math>B'C'</math> is <math>\frac{1}{6}</math> the altitude of <math>\triangle ABC</math>.
 +
The ratio of the area of <math>\triangle A'B'C'</math> to the area of <math>\triangle ABC</math> is
  
$\textbf{(A)}\ \frac{1}{36}\qquad
+
<math>\textbf{(A)}\ \frac{1}{36}\qquad
 
\textbf{(B)}\ \frac{1}{6}\qquad
 
\textbf{(B)}\ \frac{1}{6}\qquad
 
\textbf{(C)}\ \frac{1}{4}\qquad
 
\textbf{(C)}\ \frac{1}{4}\qquad
 
\textbf{(D)}\ \frac{\sqrt{3}}{4}\qquad
 
\textbf{(D)}\ \frac{\sqrt{3}}{4}\qquad
\textbf{(E)}\ \frac{9+8\sqrt{3}}{36} $    
+
\textbf{(E)}\ \frac{9+8\sqrt{3}}{36} </math>    
 
    
 
    
 
[[1988 AHSME Problems/Problem 16|Solution]]
 
[[1988 AHSME Problems/Problem 16|Solution]]
Line 231: Line 287:
 
==Problem 17==
 
==Problem 17==
  
If $|x| + x + y = 10$ and $x + |y| - y = 12$, find $x + y$
+
If <math>|x| + x + y = 10</math> and <math>x + |y| - y = 12</math>, find <math>x + y</math>
  
$\textbf{(A)}\ -2\qquad
+
<math>\textbf{(A)}\ -2\qquad
 
\textbf{(B)}\ 2\qquad
 
\textbf{(B)}\ 2\qquad
 
\textbf{(C)}\ \frac{18}{5}\qquad
 
\textbf{(C)}\ \frac{18}{5}\qquad
 
\textbf{(D)}\ \frac{22}{3}\qquad
 
\textbf{(D)}\ \frac{22}{3}\qquad
\textbf{(E)}\ 22    $
+
\textbf{(E)}\ 22    </math>
 
    
 
    
 
[[1988 AHSME Problems/Problem 17|Solution]]
 
[[1988 AHSME Problems/Problem 17|Solution]]
Line 244: Line 300:
  
 
At the end of a professional bowling tournament, the top 5 bowlers have a playoff.  
 
At the end of a professional bowling tournament, the top 5 bowlers have a playoff.  
First #5 bowls #4. The loser receives $5$th prize and the winner bowls #3 in another game.  
+
First #5 bowls #4. The loser receives <math>5</math>th prize and the winner bowls #3 in another game.  
The loser of this game receives $4$th prize and the winner bowls #2.  
+
The loser of this game receives <math>4</math>th prize and the winner bowls #2.  
The loser of this game receives $3$rd prize and the winner bowls #1.  
+
The loser of this game receives <math>3</math>rd prize and the winner bowls #1.  
 
The winner of this game gets 1st prize and the loser gets 2nd prize.  
 
The winner of this game gets 1st prize and the loser gets 2nd prize.  
 
In how many orders can bowlers #1 through #5 receive the prizes?
 
In how many orders can bowlers #1 through #5 receive the prizes?
  
\textbf{(A)}\ 10\qquad
+
<math>\textbf{(A)}\ 10\qquad
 
\textbf{(B)}\ 16\qquad
 
\textbf{(B)}\ 16\qquad
 
\textbf{(C)}\ 24\qquad
 
\textbf{(C)}\ 24\qquad
 
\textbf{(D)}\ 120\qquad
 
\textbf{(D)}\ 120\qquad
\textbf{(E)}\ \text{none of these}    
+
\textbf{(E)}\ \text{none of these}   </math> 
 
    
 
    
 
[[1988 AHSME Problems/Problem 18|Solution]]
 
[[1988 AHSME Problems/Problem 18|Solution]]
Line 262: Line 318:
 
Simplify
 
Simplify
  
$\frac{bx(a^2x^2 + 2a^2y^2 + b^2y^2) + ay(a^2x^2 + 2b^2x^2 + b^2y^2)}{bx + ay}$
+
<math>\frac{bx(a^2x^2 + 2a^2y^2 + b^2y^2) + ay(a^2x^2 + 2b^2x^2 + b^2y^2)}{bx + ay}</math>
  
$\textbf{(A)}\ a^2x^2 + b^2y^2\qquad
+
<math>\textbf{(A)}\ a^2x^2 + b^2y^2\qquad
 
\textbf{(B)}\ (ax + by)^2\qquad
 
\textbf{(B)}\ (ax + by)^2\qquad
 
\textbf{(C)}\ (ax + by)(bx + ay)\qquad\\
 
\textbf{(C)}\ (ax + by)(bx + ay)\qquad\\
 
\textbf{(D)}\ 2(a^2x^2+b^2y^2)\qquad
 
\textbf{(D)}\ 2(a^2x^2+b^2y^2)\qquad
\textbf{(E)}\ (bx+ay)^2$    
+
\textbf{(E)}\ (bx+ay)^2</math>    
 
    
 
    
 
[[1988 AHSME Problems/Problem 19|Solution]]
 
[[1988 AHSME Problems/Problem 19|Solution]]
Line 274: Line 330:
 
==Problem 20==
 
==Problem 20==
  
In one of the adjoining figures a square of side $2$ is dissected into four pieces so that $E$ and $F$ are the midpoints
+
In one of the adjoining figures a square of side <math>2</math> is dissected into four pieces so that <math>E</math> and <math>F</math> are the midpoints
of opposite sides and $AG$ is perpendicular to $BF$. These four pieces can then be reassembled into a rectangle as shown  
+
of opposite sides and <math>AG</math> is perpendicular to <math>BF</math>. These four pieces can then be reassembled into a rectangle as shown  
in the second figure. The ratio of height to base, $XY / YZ$, in this rectangle is   
+
in the second figure. The ratio of height to base, <math>XY / YZ</math>, in this rectangle is   
  
 
<asy>
 
<asy>
Line 404: Line 460:
 
<math>\textbf{(A)}\ AB=3, CD=1\qquad
 
<math>\textbf{(A)}\ AB=3, CD=1\qquad
 
\textbf{(B)}\ AB=5, CD=2\qquad
 
\textbf{(B)}\ AB=5, CD=2\qquad
\textbf{(C)}\ AB=7, CD=3\qquad
+
\textbf{(C)}\ AB=7, CD=3\qquad\\
 
\textbf{(D)}\ AB=9, CD=4\qquad
 
\textbf{(D)}\ AB=9, CD=4\qquad
 
\textbf{(E)}\ AB=11, CD=5  </math>
 
\textbf{(E)}\ AB=11, CD=5  </math>
Line 413: Line 469:
  
 
An unfair coin has probability <math>p</math> of coming up heads on a single toss.  
 
An unfair coin has probability <math>p</math> of coming up heads on a single toss.  
Let <math>w</math> be the probability that, in <math>5</math> independent toss of this coin, heads come up exactly <math>3</math> times. If <math>w = 144 / 625</math>, then  
+
Let <math>w</math> be the probability that, in <math>5</math> independent toss of this coin,  
 +
heads come up exactly <math>3</math> times. If <math>w = 144 / 625</math>, then  
  
<math>\textbf{(A)}\ p\text{ must be }2/5
+
<math>\textbf{(A)}\ p\text{ must be }\tfrac{2}{5}\qquad
\textbf{(B)}\ p\text{ must be }3/5
+
\textbf{(B)}\ p\text{ must be }\tfrac{3}{5}\qquad\\
\textbf{(C)}\ p\text{ must be greater than }3/5
+
\textbf{(C)}\ p\text{ must be greater than }\tfrac{3}{5}\qquad
\textbf{(D)}\ p\text{ is not uniquely determined}
+
\textbf{(D)}\ p\text{ is not uniquely determined}\qquad\\
\textbf{(E)}\ \text{there is no value of }p\text{ for which }w = 144/625 </math>     
+
\textbf{(E)}\ \text{there is no value of } p \text{ for which }w =\tfrac{144}{625}</math>     
 
    
 
    
 
[[1988 AHSME Problems/Problem 28|Solution]]
 
[[1988 AHSME Problems/Problem 28|Solution]]
Line 447: Line 504:
 
\textbf{(C)}\ \text{3, 4, 5 or 6}\qquad\\
 
\textbf{(C)}\ \text{3, 4, 5 or 6}\qquad\\
 
\textbf{(D)}\ \text{more than 6 but finitely many}\qquad\\
 
\textbf{(D)}\ \text{more than 6 but finitely many}\qquad\\
\textbf{(E)}\ \text{infinitely many}</math>  
+
\textbf{(E) }\infty</math>  
  
 
[[1988 AHSME Problems/Problem 30|Solution]]
 
[[1988 AHSME Problems/Problem 30|Solution]]
 +
 +
 +
 +
== See also ==
 +
 +
* [[AMC 12 Problems and Solutions]]
 +
* [[Mathematics competition resources]]
 +
 +
{{AHSME box|year=1988|before=[[1987 AHSME]]|after=[[1989 AHSME]]}} 
 +
 +
{{MAA Notice}}

Latest revision as of 13:45, 19 February 2020

1988 AHSME (Answer Key)
Printable versions: WikiAoPS ResourcesPDF

Instructions

  1. This is a 30-question, multiple choice test. Each question is followed by answers marked A, B, C, D and E. Only one of these is correct.
  2. You will receive 5 points for each correct answer, 2 points for each problem left unanswered, and 0 points for each incorrect answer.
  3. No aids are permitted other than scratch paper, graph paper, ruler, compass, protractor and erasers.
  4. Figures are not necessarily drawn to scale.
  5. You will have 90 minutes working time to complete the test.
1 2 3 4 5 6 7 8 9 10 11 12 13 14 15 16 17 18 19 20 21 22 23 24 25 26 27 28 29 30

Problem 1

$\sqrt{8}+\sqrt{18}=$

$\text{(A)}\ \sqrt{20} \qquad  \text{(B)}\ 2(\sqrt{2}+\sqrt{3}) \qquad  \text{(C)}\ 7 \qquad  \text{(D)}\ 5\sqrt{2}\qquad \text{(E)}\ 2\sqrt{13}$

Solution

Problem 2

Triangles $ABC$ and $XYZ$ are similar, with $A$ corresponding to $X$ and $B$ to $Y$. If $AB=3, BC=4$, and $XY=5$, then $YZ$ is:

$\text{(A)}\ 3\frac{3}{4} \qquad  \text{(B)}\ 6 \qquad  \text{(C)}\ 6\frac{1}{4} \qquad  \text{(D)}\ 6\frac{2}{3} \qquad  \text{(E)}\ 8$

Solution

Problem 3

[asy] draw((0,0)--(1,0)--(1,4)--(0,4)--(0,0)--(0,1)--(-1,1)--(-1,2)); draw((-1,2)--(0,2)--(0,4)--(-1,4)--(-1,5)--(1,5)--(1,6)--(0,6)); draw((0,6)--(0,5)--(3,5)--(3,6)--(4,6)--(4,2)--(5,2)); draw((5,2)--(5,1)--(1,1)--(3,1)--(3,0)--(4,0)--(4,1)); draw((1,4)--(3,4)--(3,2)--(1,2)--(4,2)--(3,2)--(3,6)); draw((3,6)--(4,6)--(4,5)--(5,5)--(5,4)--(4,4)); [/asy]

Four rectangular paper strips of length $10$ and width $1$ are put flat on a table and overlap perpendicularly as shown. How much area of the table is covered?

$\text{(A)}\ 36 \qquad  \text{(B)}\ 40 \qquad  \text{(C)}\ 44 \qquad  \text{(D)}\ 98 \qquad  \text{(E)}\ 100$

Solution


Problem 4

The slope of the line $\frac{x}{3} + \frac{y}{2} = 1$ is

$\textbf{(A)}\ -\frac{3}{2}\qquad \textbf{(B)}\ -\frac{2}{3}\qquad \textbf{(C)}\ \frac{1}{3}\qquad \textbf{(D)}\ \frac{2}{3}\qquad \textbf{(E)}\ \frac{3}{2}$

Solution

Problem 5

If $b$ and $c$ are constants and $(x + 2)(x + b) = x^2 + cx + 6$, then $c$ is

$\textbf{(A)}\ -5\qquad \textbf{(B)}\ -3\qquad \textbf{(C)}\ -1\qquad \textbf{(D)}\ 3\qquad \textbf{(E)}\ 5$

Solution

Problem 6

A figure is an equiangular parallelogram if and only if it is a

$\textbf{(A)}\ \text{rectangle}\qquad \textbf{(B)}\ \text{regular polygon}\qquad \textbf{(C)}\ \text{rhombus}\qquad \textbf{(D)}\ \text{square}\qquad \textbf{(E)}\ \text{trapezoid}$

Solution

Problem 7

Estimate the time it takes to send $60$ blocks of data over a communications channel if each block consists of $512$ "chunks" and the channel can transmit $120$ chunks per second.

$\textbf{(A)}\ 0.04 \text{ seconds}\qquad \textbf{(B)}\ 0.4 \text{ seconds}\qquad \textbf{(C)}\ 4 \text{ seconds}\qquad \textbf{(D)}\ 4\text{ minutes}\qquad \textbf{(E)}\ 4\text{ hours}$

Solution

Problem 8

If $\frac{b}{a} = 2$ and $\frac{c}{b} = 3$, what is the ratio of $a + b$ to $b + c$?

$\textbf{(A)}\ \frac{1}{3}\qquad \textbf{(B)}\ \frac{3}{8}\qquad \textbf{(C)}\ \frac{3}{5}\qquad \textbf{(D)}\ \frac{2}{3}\qquad \textbf{(E)}\ \frac{3}{4}$

Solution

Problem 9

[asy] defaultpen(linewidth(0.7)+fontsize(10)); pair A=(0,0), B=(16,0), C=(16,16), D=(0,16), E=(32,0), F=(48,0), G=(48,16), H=(32,16), I=(0,8), J=(10,8), K=(10,16), L=(32,6), M=(40,6), N=(40,16); draw(A--B--C--D--A^^E--F--G--H--E^^I--J--K^^L--M--N); label("S", (18,8)); label("S", (50,8)); label("Figure 1", (A+B)/2, S); label("Figure 2", (E+F)/2, S); label("10'", (I+J)/2, S); label("8'", (12,12)); label("8'", (L+M)/2, S); label("10'", (42,11)); label("table", (5,12)); label("table", (36,11)); [/asy]

An $8' \times 10'$ table sits in the corner of a square room, as in Figure $1$ below. The owners desire to move the table to the position shown in Figure $2$. The side of the room is $S$ feet. What is the smallest integer value of $S$ for which the table can be moved as desired without tilting it or taking it apart?

$\textbf{(A)}\ 11\qquad \textbf{(B)}\ 12\qquad \textbf{(C)}\ 13\qquad \textbf{(D)}\ 14\qquad \textbf{(E)}\ 15$

Solution

Problem 10

In an experiment, a scientific constant $C$ is determined to be $2.43865$ with an error of at most $\pm 0.00312$. The experimenter wishes to announce a value for $C$ in which every digit is significant. That is, whatever $C$ is, the announced value must be the correct result when $C$ is rounded to that number of digits. The most accurate value the experimenter can announce for $C$ is

$\textbf{(A)}\ 2\qquad \textbf{(B)}\ 2.4\qquad \textbf{(C)}\ 2.43\qquad \textbf{(D)}\ 2.44\qquad \textbf{(E)}\ 2.439$

Solution

Problem 11

[asy] defaultpen(linewidth(0.7)+fontsize(10)); pair A=(5,0), B=(7,0), C=(10,0), D=(13,0), E=(16,0); pair F=(4,3), G=(5,3), H=(7,3), I=(10,3), J=(12,3); dot(A); dot(B); dot(C); dot(D); dot(E); dot(F); dot(G); dot(H); dot(I); dot(J); draw((0,0)--(18,0)^^(0,3)--(18,3)); draw((0,0)--(0,.5)^^(5,0)--(5,.5)^^(10,0)--(10,.5)^^(15,0)--(15,.5)); draw((0,3)--(0,2.5)^^(5,3)--(5,2.5)^^(10,3)--(10,2.5)^^(15,3)--(15,2.5)); draw((1,0)--(1,.2)^^(2,0)--(2,.2)^^(3,0)--(3,.2)^^(4,0)--(4,.2)^^(6,0)--(6,.2)^^(7,0)--(7,.2)^^(8,0)--(8,.2)^^(9,0)--(9,.2)^^(10,0)--(10,.2)^^(11,0)--(11,.2)^^(12,0)--(12,.2)^^(13,0)--(13,.2)^^(14,0)--(14,.2)^^(16,0)--(16,.2)^^(17,0)--(17,.2)^^(18,0)--(18,.2)); draw((1,3)--(1,2.8)^^(2,3)--(2,2.8)^^(3,3)--(3,2.8)^^(4,3)--(4,2.8)^^(6,3)--(6,2.8)^^(7,3)--(7,2.8)^^(8,3)--(8,2.8)^^(9,3)--(9,2.8)^^(10,3)--(10,2.8)^^(11,3)--(11,2.8)^^(12,3)--(12,2.8)^^(13,3)--(13,2.8)^^(14,3)--(14,2.8)^^(16,3)--(16,2.8)^^(17,3)--(17,2.8)^^(18,3)--(18,2.8)); label("A", A, S); label("B", B, S); label("C", C, S); label("D", D, S); label("E", E, S); label("A", F, N); label("B", G, N); label("C", H, N); label("D", I, N); label("E", J, N); label("1970", (0,3), W); label("1980", (0,0), W); label("0", (0,1.5)); label("50", (5,1.5)); label("100", (10,1.5)); label("150", (15,1.5)); label("Population in thousands", (9,-3)); [/asy]

On each horizontal line in the figure below, the five large dots indicate the populations of cities $A, B, C, D$ and $E$ in the year indicated. Which city had the greatest percentage increase in population from $1970$ to $1980$?

$\textbf{(A)}\ A\qquad \textbf{(B)}\ B\qquad \textbf{(C)}\ C\qquad \textbf{(D)}\ D\qquad \textbf{(E)}\ E$

Solution

Problem 12

Each integer $1$ through $9$ is written on a separate slip of paper and all nine slips are put into a hat. Jack picks one of these slips at random and puts it back. Then Jill picks a slip at random. Which digit is most likely to be the units digit of the sum of Jack's integer and Jill's integer?

$\textbf{(A)}\ 0\qquad \textbf{(B)}\ 1\qquad \textbf{(C)}\ 8\qquad \textbf{(D)}\ 9\qquad \textbf{(E)}\ \text{each digit is equally likely}$

Solution

Problem 13

If $\sin(x)= 3\cos(x)$ then what is $\sin(x) \cdot \cos(x)$?

$\textbf{(A)}\ \frac{1}{6}\qquad \textbf{(B)}\ \frac{1}{5}\qquad \textbf{(C)}\ \frac{2}{9}\qquad \textbf{(D)}\ \frac{1}{4}\qquad \textbf{(E)}\ \frac{3}{10}$

Solution

Problem 14

For any real number a and positive integer k, define

${a \choose k} = \frac{a(a-1)(a-2)\cdots(a-(k-1))}{k(k-1)(k-2)\cdots(2)(1)}$

What is

${-\frac{1}{2} \choose 100} \div {\frac{1}{2} \choose 100}$?

$\textbf{(A)}\ -199\qquad \textbf{(B)}\ -197\qquad \textbf{(C)}\ -1\qquad \textbf{(D)}\ 197\qquad \textbf{(E)}\ 199$

Solution

Problem 15

If $a$ and $b$ are integers such that $x^2 - x - 1$ is a factor of $ax^3 + bx^2 + 1$, then $b$ is

$\textbf{(A)}\ -2\qquad \textbf{(B)}\ -1\qquad \textbf{(C)}\ 0\qquad \textbf{(D)}\ 1\qquad \textbf{(E)}\ 2$

Solution

Problem 16

[asy] defaultpen(linewidth(0.7)+fontsize(10)); pair H=origin, B=(1,-(1/sqrt(3))), C=(-1,-(1/sqrt(3))), A=(0,(2/sqrt(3))), E=(2,-(2/sqrt(3))), F=(-2,-(2/sqrt(3))), D=(0,(4/sqrt(3))); draw(A--B--C--A^^D--E--F--D); label("A'", A, N); label("B'", B, SE); label("C'", C, SW); label("A", D, E); label("B", E, E); label("C", F, W); [/asy]

$ABC$ and $A'B'C'$ are equilateral triangles with parallel sides and the same center, as in the figure. The distance between side $BC$ and side $B'C'$ is $\frac{1}{6}$ the altitude of $\triangle ABC$. The ratio of the area of $\triangle A'B'C'$ to the area of $\triangle ABC$ is

$\textbf{(A)}\ \frac{1}{36}\qquad \textbf{(B)}\ \frac{1}{6}\qquad \textbf{(C)}\ \frac{1}{4}\qquad \textbf{(D)}\ \frac{\sqrt{3}}{4}\qquad \textbf{(E)}\ \frac{9+8\sqrt{3}}{36}$

Solution

Problem 17

If $|x| + x + y = 10$ and $x + |y| - y = 12$, find $x + y$

$\textbf{(A)}\ -2\qquad \textbf{(B)}\ 2\qquad \textbf{(C)}\ \frac{18}{5}\qquad \textbf{(D)}\ \frac{22}{3}\qquad \textbf{(E)}\ 22$

Solution

Problem 18

At the end of a professional bowling tournament, the top 5 bowlers have a playoff. First #5 bowls #4. The loser receives $5$th prize and the winner bowls #3 in another game. The loser of this game receives $4$th prize and the winner bowls #2. The loser of this game receives $3$rd prize and the winner bowls #1. The winner of this game gets 1st prize and the loser gets 2nd prize. In how many orders can bowlers #1 through #5 receive the prizes?

$\textbf{(A)}\ 10\qquad \textbf{(B)}\ 16\qquad \textbf{(C)}\ 24\qquad \textbf{(D)}\ 120\qquad \textbf{(E)}\ \text{none of these}$

Solution

Problem 19

Simplify

$\frac{bx(a^2x^2 + 2a^2y^2 + b^2y^2) + ay(a^2x^2 + 2b^2x^2 + b^2y^2)}{bx + ay}$

$\textbf{(A)}\ a^2x^2 + b^2y^2\qquad \textbf{(B)}\ (ax + by)^2\qquad \textbf{(C)}\ (ax + by)(bx + ay)\qquad\\ \textbf{(D)}\ 2(a^2x^2+b^2y^2)\qquad \textbf{(E)}\ (bx+ay)^2$

Solution

Problem 20

In one of the adjoining figures a square of side $2$ is dissected into four pieces so that $E$ and $F$ are the midpoints of opposite sides and $AG$ is perpendicular to $BF$. These four pieces can then be reassembled into a rectangle as shown in the second figure. The ratio of height to base, $XY / YZ$, in this rectangle is

[asy] defaultpen(linewidth(0.7)+fontsize(10)); pair A=(0,1), B=(0,-1), C=(2,-1), D=(2,1), E=(1,-1), F=(1,1), G=(.8,.6); pair X=(4,sqrt(5)), Y=(4,-sqrt(5)), Z=(4+2/sqrt(5),-sqrt(5)), W=(4+2/sqrt(5),sqrt(5)), T=(4,0), U=(4+2/sqrt(5),-4/sqrt(5)), V=(4+2/sqrt(5),1/sqrt(5)); draw(A--B--C--D--A^^B--F^^E--D^^A--G^^rightanglemark(A,G,F)); draw(X--Y--Z--W--X^^T--V--X^^Y--U); label("A", A, NW); label("B", B, SW); label("C", C, SE); label("D", D, NE); label("E", E, S); label("F", F, N); label("G", G, E); label("X", X, NW); label("Y", Y, SW); label("Z", Z, SE); label("W", W, NE); [/asy]

$\textbf{(A)}\ 4\qquad \textbf{(B)}\ 1+2\sqrt{3}\qquad \textbf{(C)}\ 2\sqrt{5}\qquad \textbf{(D)}\ \frac{8+4\sqrt{3}}{3}\qquad \textbf{(E)}\ 5$

Solution

Problem 21

The complex number $z$ satisfies $z + |z| = 2 + 8i$. What is $|z|^{2}$? Note: if $z = a + bi$, then $|z| = \sqrt{a^{2} + b^{2}}$.

$\textbf{(A)}\ 68\qquad \textbf{(B)}\ 100\qquad \textbf{(C)}\ 169\qquad \textbf{(D)}\ 208\qquad \textbf{(E)}\ 289$

Solution

Problem 22

For how many integers $x$ does a triangle with side lengths $10, 24$ and $x$ have all its angles acute?

$\textbf{(A)}\ 4\qquad \textbf{(B)}\ 5\qquad \textbf{(C)}\ 6\qquad \textbf{(D)}\ 7\qquad \textbf{(E)}\ \text{more than } 7$

Solution

Problem 23

The six edges of a tetrahedron $ABCD$ measure $7, 13, 18, 27, 36$ and $41$ units. If the length of edge $AB$ is $41$, then the length of edge $CD$ is

$\textbf{(A)}\ 7\qquad \textbf{(B)}\ 13\qquad \textbf{(C)}\ 18\qquad \textbf{(D)}\ 27\qquad \textbf{(E)}\ 36$

Solution

Problem 24

An isosceles trapezoid is circumscribed around a circle. The longer base of the trapezoid is $16$, and one of the base angles is $\arcsin(.8)$. Find the area of the trapezoid.

$\textbf{(A)}\ 72\qquad \textbf{(B)}\ 75\qquad \textbf{(C)}\ 80\qquad \textbf{(D)}\ 90\qquad \textbf{(E)}\ \text{not uniquely determined}$

Solution

Problem 25

$X, Y$ and $Z$ are pairwise disjoint sets of people. The average ages of people in the sets $X, Y, Z, X \cup Y, X \cup Z$ and $Y \cup Z$ are $37, 23, 41, 29, 39.5$ and $33$ respectively. Find the average age of the people in set $X \cup Y \cup Z$.

$\textbf{(A)}\ 33\qquad \textbf{(B)}\ 33.5\qquad \textbf{(C)}\ 33.6\overline{6}\qquad \textbf{(D)}\ 33.83\overline{3}\qquad \textbf{(E)}\ 34$

Solution

Problem 26

Suppose that $p$ and $q$ are positive numbers for which

$\log_{9}(p) = \log_{12}(q) = \log_{16}(p+q)$

What is the value of $\frac{q}{p}$?

$\textbf{(A)}\ \frac{4}{3}\qquad \textbf{(B)}\ \frac{1+\sqrt{3}}{2}\qquad \textbf{(C)}\ \frac{8}{5}\qquad \textbf{(D)}\ \frac{1+\sqrt{5}}{2}\qquad \textbf{(E)}\ \frac{16}{9}$

Solution

Problem 27

In the figure, $AB \perp BC, BC \perp CD$, and $BC$ is tangent to the circle with center $O$ and diameter $AD$. In which one of the following cases is the area of $ABCD$ an integer?

[asy] defaultpen(fontsize(10pt)+linewidth(.8pt)); pair O=origin, A=(-1/sqrt(2),1/sqrt(2)), B=(-1/sqrt(2),-1), C=(1/sqrt(2),-1), D=(1/sqrt(2),-1/sqrt(2)); draw(unitcircle); dot(O); draw(A--B--C--D--A); label("$A$",A,dir(A)); label("$B$",B,dir(B)); label("$C$",C,dir(C)); label("$D$",D,dir(D)); label("$O$",O,N); [/asy]

$\textbf{(A)}\ AB=3, CD=1\qquad \textbf{(B)}\ AB=5, CD=2\qquad \textbf{(C)}\ AB=7, CD=3\qquad\\ \textbf{(D)}\ AB=9, CD=4\qquad \textbf{(E)}\ AB=11, CD=5$

Solution

Problem 28

An unfair coin has probability $p$ of coming up heads on a single toss. Let $w$ be the probability that, in $5$ independent toss of this coin, heads come up exactly $3$ times. If $w = 144 / 625$, then

$\textbf{(A)}\ p\text{ must be }\tfrac{2}{5}\qquad \textbf{(B)}\ p\text{ must be }\tfrac{3}{5}\qquad\\ \textbf{(C)}\ p\text{ must be greater than }\tfrac{3}{5}\qquad \textbf{(D)}\ p\text{ is not uniquely determined}\qquad\\ \textbf{(E)}\ \text{there is no value of } p \text{ for which }w =\tfrac{144}{625}$

Solution

Problem 29

You plot weight $(y)$ against height $(x)$ for three of your friends and obtain the points $(x_{1},y_{1}), (x_{2},y_{2}), (x_{3},y_{3})$. If $x_{1} < x_{2} < x_{3}$ and $x_{3} - x_{2} = x_{2} - x_{1}$, which of the following is necessarily the slope of the line which best fits the data? "Best fits" means that the sum of the squares of the vertical distances from the data points to the line is smaller than for any other line.

$\textbf{(A)}\ \frac{y_{3}-y_{1}}{x_{3}-x_{1}}\qquad \textbf{(B)}\ \frac{(y_{2}-y_{1})-(y_{3}-y_{2})}{x_{3}-x_{1}}\qquad\\ \textbf{(C)}\ \frac{2y_{3}-y_{1}-y_{2}}{2x_{3}-x_{1}-x_{2}}\qquad \textbf{(D)}\ \frac{y_{2}-y_{1}}{x_{2}-x_{1}}+\frac{y_{3}-y_{2}}{x_{3}-x_{2}}\qquad\\ \textbf{(E)}\ \text{none of these}$

Solution

Problem 30

Let $f(x) = 4x - x^{2}$. Give $x_{0}$, consider the sequence defined by $x_{n} = f(x_{n-1})$ for all $n \ge 1$. For how many real numbers $x_{0}$ will the sequence $x_{0}, x_{1}, x_{2}, \ldots$ take on only a finite number of different values?

$\textbf{(A)}\ \text{0}\qquad \textbf{(B)}\ \text{1 or 2}\qquad \textbf{(C)}\ \text{3, 4, 5 or 6}\qquad\\ \textbf{(D)}\ \text{more than 6 but finitely many}\qquad\\ \textbf{(E) }\infty$

Solution


See also

1988 AHSME (ProblemsAnswer KeyResources)
Preceded by
1987 AHSME
Followed by
1989 AHSME
1 2 3 4 5 6 7 8 9 10 11 12 13 14 15 16 17 18 19 20 21 22 23 24 25 26 27 28 29 30
All AHSME Problems and Solutions


The problems on this page are copyrighted by the Mathematical Association of America's American Mathematics Competitions. AMC logo.png